Question

A circular loop of wire with radius 0.0250 m and resistance 0.340 is in a region of spatially uniform magnetic field, as show
0 0
Add a comment Improve this question Transcribed image text
Answer #1

Sel- Given - r = 0.025 m -R = 0.3402 x Blt) = 0.45/s33 Х X X (0) 5 = ? when B=1.5T from, Faradays Law of Elechongnetisms a tfem fransdogs law, Es de dt d [0.443 x 1 (0.025 dt 0.002 356 +2 2.356 ta v my Curing ) t = 1.55 so E = 2.3 56 (1.ss} 5.66 mV

Add a comment
Know the answer?
Add Answer to:
A circular loop of wire with radius 0.0250 m and resistance 0.340 is in a region...
Your Answer:

Post as a guest

Your Name:

What's your source?

Earn Coins

Coins can be redeemed for fabulous gifts.

Not the answer you're looking for? Ask your own homework help question. Our experts will answer your question WITHIN MINUTES for Free.
Similar Homework Help Questions
  • A circular loop of wire with radius 0.0290 m and resistance 0.370Ω

    A circular loop of wire with radius 0.0290 m and resistance 0.370Ω is in a region of spatially uniform magnetic field, as shown in the following figure(Figure 1). The magnetic field is directed into the plane of the figure. At t = 0, B = 0. The magnetic field then begins increasing, with B(t) = (0.400 T/s3)t3 Part A What is the current in the loop (magnitude) at the instant when B = 1.29 T?  

  • A circular loop of wire with radius 0.0270 m and resistance 0.320 Ω

    A circular loop of wire with radius 0.0270 m and resistance 0.320 Ω is in a region of spatially uniform magnetic field, as shown in the following figure(Figure 1). The magnetic field is directed into the plane of the figure. At t = 0, B = 0. The magnetic field then begins increasing, with B(t) =( 0.400 T/s3)t3 .Part A What is the current in the loop (magnitude) at the instant when B = 1.38 T? Part B What is the direction of the...

  • A circular loop of wire with radius r= 0.0480m and reistance R = 0.160 Ω is in a region of spatially uniform magnetic field

    A circular loop of wire with radius r= 0.0480m and reistance R = 0.160 Ω is in a region of spatially uniform magnetic field, as shown in the figure. The magnetic field is directed out of the plane of the figure. The magnetic field has an initial value of 8.00 T and is decreasing dB/ dt = -0.680 T a) Is the induced current in the loop clockwise or counterclockwise? b) What is the rate at which electrical energy is being dissipated...

  • A circular loop of wire of resistance - 0.500 and radius 8.10 cm is in a...

    A circular loop of wire of resistance - 0.500 and radius 8.10 cm is in a uniform magnetic field directed out of the page as in the figure below. A clockwise current of 120 mA is induced in the loop. (a) is the magnetic field increasing or decreasing in time? decreasing (b) Find the rate at which the field is changing with time.

  • Circular loop of wire

    A circular loop of wire is in a region of spatially uniformmagnetic field. The magnetic field is directed into the plane ofthe figure.Determine the direction of the induced current in the loop when isincreasing.Determine the direction of the induced current in the loop when isdecreasing.Determine the direction of the induced current in the loop when isconstant with value .

  • A single-turn circular loop of wire of radius 68 mm lies in a plane perpendicular to...

    A single-turn circular loop of wire of radius 68 mm lies in a plane perpendicular to a spatially uniform magnetic field. During a 0.1 s time interval, the magnitude of the field increases uniformly from 200 to 300 mT. (a) Determine the emf induced in the loop. Submit Answer Tries 0/3 (b) If the magnetic field is directed out of the page (and you're looking from above!), what is the direction of the current induced in the loop? Why? Submission...

  • 24 A circular loop of wire is in a region of spatially uniform magnetic field directed...

    24 A circular loop of wire is in a region of spatially uniform magnetic field directed into the page. Determine the direction of the induced current (as viewed from above) when the magnitude of the magnetic field is increasing. a. Clockwise b. Counterclockwise c. No current flows. 25 An electron enters a region where the uniform magnetic field strength is 4.0 T at a velocity of 4.88 x 10^m/s perpendicular to the field. Determine the radius of gyration of the...

  • Q1. (25 points) A circular loop of wire of resistance R = 1 N and radius...

    Q1. (25 points) A circular loop of wire of resistance R = 1 N and radius r = 15 cm is in a uniform magnetic field directed out of the page as shown in the figure. a) (7 points) If a clockwise current of I = 8 mA is induced in the loop, is the magnetic field increasing or decreasing in time? b) (8 points) Find the induced emf in the loop. c) (10 points) Find the rate at which...

  • Q1. (25 points) A circular loop of wire of resistance R = 4 N and radius...

    Q1. (25 points) A circular loop of wire of resistance R = 4 N and radius r = 30 cm is in a uniform magnetic field directed out of the page as shown in the figure. a) (7 points) If a clockwise current of 1 = 20 mA is induced in the loop, is the magnetic field increasing or decreasing in time? b) (8 points) Find the induced emf in the loop. c) (10 points) Find the rate at which...

  • (11%) Problem 8: A circular wire loop of radius r = 0.35 m and resistance R=...

    (11%) Problem 8: A circular wire loop of radius r = 0.35 m and resistance R= 11 12 rotates about a shaft through its diameter at a constant rate of f= 5.5 Hz in a uniform B = 0.21-T magnetic field directed perpendicular to the rotation axis. The plane of the loop is perpendicular to the magnetic field at time t = 0. 4 17% Part (a) Select the correct expression for the time-dependent magnetic flux through the loop. 17%...

ADVERTISEMENT
Free Homework Help App
Download From Google Play
Scan Your Homework
to Get Instant Free Answers
Need Online Homework Help?
Ask a Question
Get Answers For Free
Most questions answered within 3 hours.
ADVERTISEMENT
ADVERTISEMENT
ADVERTISEMENT